Mathcenter Forum  

Go Back   Mathcenter Forum > คณิตศาสตร์โอลิมปิก และอุดมศึกษา > อสมการ
สมัครสมาชิก คู่มือการใช้ รายชื่อสมาชิก ปฏิทิน ข้อความวันนี้

ตั้งหัวข้อใหม่ Reply
 
เครื่องมือของหัวข้อ ค้นหาในหัวข้อนี้
  #16  
Old 03 ตุลาคม 2009, 20:26
nooonuii nooonuii ไม่อยู่ในระบบ
ผู้พิทักษ์กฎทั่วไป
 
วันที่สมัครสมาชิก: 25 พฤษภาคม 2001
ข้อความ: 6,408
nooonuii is on a distinguished road
Default

ก่อนอื่นต้องบอกก่อนว่าที่ผมใช้ภาษาอังกฤษบ้างไทยบ้าง

เป็นเพราะผมตอบกระทู้ต่างสถานที่กันครับ

ถ้าผมอยู่ที่บ้านผมก็ใช้ภาษาไทยได้

แต่ถ้าอยู่ที่มหาวิทยาลัยผมต้องใช้ภาษาอังกฤษ

เพราะเครื่องคอมฯไม่มีแป้นพิมพ์ภาษาไทยครับ

-----------------------------------------------------

ข้อนี้ trick อยู่ที่การจัดรูปก่อนใช้ AM-GM ครับ

อสมการแรกลองจัดรูปให้ได้แบบนี้ครับ

$n(n+1)^{\frac{1}{n}}\leq (1+1) + (1+\dfrac{1}{2})+\cdots + (1+\dfrac{1}{n})$

แล้วจะเห็นอสมการ AM-GM ลอยมาเลยครับ

อสมการที่สองก็ใช้ trick เดียวกัน
__________________
site:mathcenter.net คำค้น
ตอบพร้อมอ้างอิงข้อความนี้
  #17  
Old 04 ตุลาคม 2009, 14:10
LightLucifer's Avatar
LightLucifer LightLucifer ไม่อยู่ในระบบ
กระบี่ธรรมชาติ
 
วันที่สมัครสมาชิก: 25 กันยายน 2008
ข้อความ: 2,352
LightLucifer is on a distinguished road
Default

อีกข้อครับๆ ข้อนี้ไม่กำหนดวิธีทำ

$Let \ a_1,a_2,...,a_k \ be \ real \ number \ satisfying \ the \ following \ two \ condition$
$(i) \ 0\leqslant a_1\leqslant a_2\leqslant ... \leqslant a_k$
$(ii) \ a_1+a_2+...+a_k=1$
$Prove \ that \ \frac{a_1+a_2+...+a_n}{n}\leqslant \frac{1}{k} \ \ \ \ \ \ \ \ \ \ for \ n = 1,2,3,...,k$
__________________
เหนือฟ้ายังมีฟ้าแต่เหนือข้าต้องไม่มีใคร

ปีกขี้ผื้งของปลอมงั้นสินะ


...โลกนี้โหดร้ายจริงๆ มันให้ความสุขกับเรา แล้วสุดท้าย มันก็เอาคืนไป...
ตอบพร้อมอ้างอิงข้อความนี้
  #18  
Old 04 ตุลาคม 2009, 21:27
nooonuii nooonuii ไม่อยู่ในระบบ
ผู้พิทักษ์กฎทั่วไป
 
วันที่สมัครสมาชิก: 25 พฤษภาคม 2001
ข้อความ: 6,408
nooonuii is on a distinguished road
Default

ทำตรงๆครับ เปลี่ยนเป็นพิสูจน์อันนี้แทน

$\dfrac{a_1+\cdots a_n}{n}\leq \dfrac{a_1+\cdots+a_k}{k}$

เมื่อจัดรูปแล้วจะได้

$n(a_{n+1}+\cdots+a_k)\geq (k-n)(a_1+\cdots +a_n)$

คราวนี้ลองใช้ประโยชน์จากเงื่อนไขแรกครับ
__________________
site:mathcenter.net คำค้น
ตอบพร้อมอ้างอิงข้อความนี้
  #19  
Old 19 ตุลาคม 2009, 20:59
LightLucifer's Avatar
LightLucifer LightLucifer ไม่อยู่ในระบบ
กระบี่ธรรมชาติ
 
วันที่สมัครสมาชิก: 25 กันยายน 2008
ข้อความ: 2,352
LightLucifer is on a distinguished road
Default

ช่วยข้อนี้ด้วยครับๆ

ให้ $S=a_1+a_2+...+a_n$ จงพิสูจน์ว่า
$\sum_{i = 1}^{n} \frac{a_i}{S-a_i}\geqslant \frac{n}{n-1}$
ผมทำได้แต่ $\sum_{i = 1}^{n} \frac{S-a_i}{a_i}\geqslant (n)(n-1)$ อ่ะครับ
__________________
เหนือฟ้ายังมีฟ้าแต่เหนือข้าต้องไม่มีใคร

ปีกขี้ผื้งของปลอมงั้นสินะ


...โลกนี้โหดร้ายจริงๆ มันให้ความสุขกับเรา แล้วสุดท้าย มันก็เอาคืนไป...
ตอบพร้อมอ้างอิงข้อความนี้
  #20  
Old 19 ตุลาคม 2009, 23:17
nooonuii nooonuii ไม่อยู่ในระบบ
ผู้พิทักษ์กฎทั่วไป
 
วันที่สมัครสมาชิก: 25 พฤษภาคม 2001
ข้อความ: 6,408
nooonuii is on a distinguished road
Default

อ้างอิง:
ข้อความเดิมเขียนโดยคุณ LightLucifer View Post
ช่วยข้อนี้ด้วยครับๆ

ให้ $S=a_1+a_2+...+a_n$ จงพิสูจน์ว่า
$\sum_{i = 1}^{n} \frac{a_i}{S-a_i}\geqslant \frac{n}{n-1}$
ผมทำได้แต่ $\sum_{i = 1}^{n} \frac{S-a_i}{a_i}\geqslant (n)(n-1)$ อ่ะครับ
First Solution : Cauchy-Schwarz inequality (twice!)

$LHS=\sum\dfrac{a_i^2}{a_iS-a_i^2}\geq ... \geq\dfrac{n}{n-1}$

Second Solution : AM - HM inequality

$LHS + n = \sum\Big(\dfrac{a_i}{S-a_i}+1\Big)=\sum\dfrac{S}{S-a_i}=S\sum\dfrac{1}{S-a_i}$

Let $x_i=S-a_i$. Then use AM-HM inequality on $x_1,...,x_n$.
__________________
site:mathcenter.net คำค้น
ตอบพร้อมอ้างอิงข้อความนี้
  #21  
Old 20 ตุลาคม 2009, 00:01
LightLucifer's Avatar
LightLucifer LightLucifer ไม่อยู่ในระบบ
กระบี่ธรรมชาติ
 
วันที่สมัครสมาชิก: 25 กันยายน 2008
ข้อความ: 2,352
LightLucifer is on a distinguished road
Default

อ๋อ ขอบคุณมากเลยครับ
มีที่สงสัยอีกข้อครับ

ถ้า $a>1,n\in \mathbb{N} $ จงพิสูจน์ว่า
$$a+a^2+...+a^{2n}\leqslant n(a^{2n+1}+1)$$
__________________
เหนือฟ้ายังมีฟ้าแต่เหนือข้าต้องไม่มีใคร

ปีกขี้ผื้งของปลอมงั้นสินะ


...โลกนี้โหดร้ายจริงๆ มันให้ความสุขกับเรา แล้วสุดท้าย มันก็เอาคืนไป...
ตอบพร้อมอ้างอิงข้อความนี้
  #22  
Old 20 ตุลาคม 2009, 10:11
nooonuii nooonuii ไม่อยู่ในระบบ
ผู้พิทักษ์กฎทั่วไป
 
วันที่สมัครสมาชิก: 25 พฤษภาคม 2001
ข้อความ: 6,408
nooonuii is on a distinguished road
Default

อ้างอิง:
ข้อความเดิมเขียนโดยคุณ LightLucifer View Post
ถ้า $a>1,n\in \mathbb{N} $ จงพิสูจน์ว่า
$$a+a^2+...+a^{2n}\leqslant n(a^{2n+1}+1)$$
ใช้ divide and conquer ครับ

$a^k+a^{2n+1-k}\leq a^{2n+1}+1$

ทุกค่า $k=1,...,n$
__________________
site:mathcenter.net คำค้น

20 ตุลาคม 2009 20:43 : ข้อความนี้ถูกแก้ไขแล้ว 1 ครั้ง, ครั้งล่าสุดโดยคุณ nooonuii
ตอบพร้อมอ้างอิงข้อความนี้
  #23  
Old 20 ตุลาคม 2009, 17:54
LightLucifer's Avatar
LightLucifer LightLucifer ไม่อยู่ในระบบ
กระบี่ธรรมชาติ
 
วันที่สมัครสมาชิก: 25 กันยายน 2008
ข้อความ: 2,352
LightLucifer is on a distinguished road
Default

งงอ่ะครับ

แล้วตัว m มาจากไหนอ่ะครับ ??
__________________
เหนือฟ้ายังมีฟ้าแต่เหนือข้าต้องไม่มีใคร

ปีกขี้ผื้งของปลอมงั้นสินะ


...โลกนี้โหดร้ายจริงๆ มันให้ความสุขกับเรา แล้วสุดท้าย มันก็เอาคืนไป...
ตอบพร้อมอ้างอิงข้อความนี้
  #24  
Old 20 ตุลาคม 2009, 20:43
nooonuii nooonuii ไม่อยู่ในระบบ
ผู้พิทักษ์กฎทั่วไป
 
วันที่สมัครสมาชิก: 25 พฤษภาคม 2001
ข้อความ: 6,408
nooonuii is on a distinguished road
Default

อ้างอิง:
ข้อความเดิมเขียนโดยคุณ LightLucifer View Post
งงอ่ะครับ

แล้วตัว m มาจากไหนอ่ะครับ ??
$m=n$ my mistake!
__________________
site:mathcenter.net คำค้น
ตอบพร้อมอ้างอิงข้อความนี้
  #25  
Old 20 ตุลาคม 2009, 21:47
LightLucifer's Avatar
LightLucifer LightLucifer ไม่อยู่ในระบบ
กระบี่ธรรมชาติ
 
วันที่สมัครสมาชิก: 25 กันยายน 2008
ข้อความ: 2,352
LightLucifer is on a distinguished road
Default

ผมยังมองไม่ออกเลยอ่ะครับว่าจะทำยังไงต่ออ่ะรบกวนช่วยแนะนำหน่อยครับ T_T
__________________
เหนือฟ้ายังมีฟ้าแต่เหนือข้าต้องไม่มีใคร

ปีกขี้ผื้งของปลอมงั้นสินะ


...โลกนี้โหดร้ายจริงๆ มันให้ความสุขกับเรา แล้วสุดท้าย มันก็เอาคืนไป...
ตอบพร้อมอ้างอิงข้อความนี้
  #26  
Old 20 ตุลาคม 2009, 22:04
winlose's Avatar
winlose winlose ไม่อยู่ในระบบ
ลมปราณบริสุทธิ์
 
วันที่สมัครสมาชิก: 17 ตุลาคม 2008
ข้อความ: 138
winlose is on a distinguished road
Default


20 ตุลาคม 2009 22:05 : ข้อความนี้ถูกแก้ไขแล้ว 1 ครั้ง, ครั้งล่าสุดโดยคุณ winlose
ตอบพร้อมอ้างอิงข้อความนี้
  #27  
Old 21 ตุลาคม 2009, 00:14
LightLucifer's Avatar
LightLucifer LightLucifer ไม่อยู่ในระบบ
กระบี่ธรรมชาติ
 
วันที่สมัครสมาชิก: 25 กันยายน 2008
ข้อความ: 2,352
LightLucifer is on a distinguished road
Default

อ๋อขอบคุณมากครับ
__________________
เหนือฟ้ายังมีฟ้าแต่เหนือข้าต้องไม่มีใคร

ปีกขี้ผื้งของปลอมงั้นสินะ


...โลกนี้โหดร้ายจริงๆ มันให้ความสุขกับเรา แล้วสุดท้าย มันก็เอาคืนไป...
ตอบพร้อมอ้างอิงข้อความนี้
  #28  
Old 29 ตุลาคม 2009, 23:07
The jumpers's Avatar
The jumpers The jumpers ไม่อยู่ในระบบ
บัณฑิตฟ้า
 
วันที่สมัครสมาชิก: 01 เมษายน 2008
ข้อความ: 432
The jumpers is on a distinguished road
Post

อ้างอิง:
ข้อความเดิมเขียนโดยคุณ LightLucifer View Post
คือตอนนี้มกำลังมึนๆกับพวกอสมการ Holder's, Chebychev's, Minkowski's, Power mean อ่ะครับ รบกวนช่วยสอนพวกเทคนิคการพิสูจน์ของอสมการพวกนี้หน่อยนะครับ
ขอข้อนี้ก่อนนะครับ

กำหนดให้ $a,b\geqslant0$ จงพิสูจน์ว่า
$$(a+b)(a^3+b^3)(a^7+b^7)\leqslant 4(a^{11}+b^{11})$$
Rearrangement Inequality
\[(a^{11}+b^{11}-a^{10}b-ab^{10})+(a^{11}+b^{11}-a^8b^3-a^3b^8)+(a^{11}+b^{11}-a^7b^4-a^4b^7)\geqslant 0\]
(เทคนิค:ทำย้อนกลับ)
ตอบพร้อมอ้างอิงข้อความนี้
  #29  
Old 29 ตุลาคม 2009, 23:18
The jumpers's Avatar
The jumpers The jumpers ไม่อยู่ในระบบ
บัณฑิตฟ้า
 
วันที่สมัครสมาชิก: 01 เมษายน 2008
ข้อความ: 432
The jumpers is on a distinguished road
Post

อ้างอิง:
ข้อความเดิมเขียนโดยคุณ LightLucifer View Post
อ๋อ ขอบคุณมากเลยครับ
แล้วถ้าเป็นข้อนี้อ่ะครับ เอาแบบไม่ใช้โคชีอ่ะครับคิดไม่ออกอ่ะ

กำหนดให้ $a,b,c\in \mathbb{R}^+$ จงพิสูจน์ว่า
$$a+b+c\leqslant \frac{a^2+b^2}{2c}+\frac{b^2+c^2}{2b}+\frac{c^2+a^2}{2a}$$
\[\sum_{cyc}a(b+c)(b-c)^2\geqslant 0\]
ตอบพร้อมอ้างอิงข้อความนี้
  #30  
Old 29 ตุลาคม 2009, 23:37
The jumpers's Avatar
The jumpers The jumpers ไม่อยู่ในระบบ
บัณฑิตฟ้า
 
วันที่สมัครสมาชิก: 01 เมษายน 2008
ข้อความ: 432
The jumpers is on a distinguished road
Post

อ้างอิง:
ข้อความเดิมเขียนโดยคุณ LightLucifer View Post
ผมขอ Chebychev อีกข้อนะครับ ยังไม่คล่องพอเลย

ให้ $a,b,c\geqslant0$ จงพิสูจน์ว่า
$$(ab+bc+ca)(a+b+c)^4\leqslant27(a^3+b^3+c^3)^2$$
Power-mean Inequality
\[27(a^3+b^3+c^3)^2=243(\sqrt[3]{\frac{a^3+b^3+c^3}{3}} )^6\geqslant 243(\frac{a+b+c}{3})^6=(\frac{(a+b+c)^2}{3})(a+b+c)^4\geqslant (ab+bc+ca)(a+b+c)^4\]
ตอบพร้อมอ้างอิงข้อความนี้
ตั้งหัวข้อใหม่ Reply



กฎการส่งข้อความ
คุณ ไม่สามารถ ตั้งหัวข้อใหม่ได้
คุณ ไม่สามารถ ตอบหัวข้อได้
คุณ ไม่สามารถ แนบไฟล์และเอกสารได้
คุณ ไม่สามารถ แก้ไขข้อความของคุณเองได้

vB code is On
Smilies are On
[IMG] code is On
HTML code is Off
ทางลัดสู่ห้อง


เวลาที่แสดงทั้งหมด เป็นเวลาที่ประเทศไทย (GMT +7) ขณะนี้เป็นเวลา 11:02


Powered by vBulletin® Copyright ©2000 - 2024, Jelsoft Enterprises Ltd.
Modified by Jetsada Karnpracha